Bạn chưa đăng nhập. Vui lòng đăng nhập để hỏi bài
pham trung thanh

Cho a;b;c là các số thực không âm thỏa mãn \(a^2+b^2+c^2>0\)\(.CMR:\)

\(\frac{3a^2-bc}{2a^2+b^2+c^2}+\frac{3b^2-ca}{+2b^2+c^2+a^2}+\frac{3c^2-ab}{2c^2+a^2+b^2}\le\frac{3}{2}\)

vũ tiền châu
15 tháng 12 2017 lúc 21:39

Làm tạm vào đây vậy

từ gt dễ dàng => \(ab+bc+ca\le3\)

\(\Rightarrow\frac{ab}{\sqrt{c^2+3}}\le\frac{ab}{\sqrt{c^2+ab+bc+ca}}=\frac{ab}{\sqrt{\left(c+a\right)\left(c+b\right)}}\)

Áp dụng cô si ta có

\(\frac{ab}{\sqrt{\left(c+a\right)\left(c+b\right)}}\le\frac{1}{2}\left(\frac{ab}{c+a}+\frac{ab}{c+b}\right)\)

Tương tự như vậy rồi ccộng vào nhá nhok


Các câu hỏi tương tự
Nguyễn Thiều Công Thành
Xem chi tiết
Nguyễn Ngọc Khánh Ly
Xem chi tiết
Hùng Quân Mai
Xem chi tiết
Lê Tài Bảo Châu
Xem chi tiết
Thanh Tùng DZ
Xem chi tiết
Siêu Nhân Lê
Xem chi tiết
hoàng thị huyền trang
Xem chi tiết
Mo Anime
Xem chi tiết
zZz Cool Kid_new zZz
Xem chi tiết